DeanX academy
Demo test
Urology, surgical gastroenterology, Medical gastroenterology, neurosurgery , oncosurgery
Download the app
https://play.google.com/store/apps/details?id=com.shakti.deansacademy
Neurosurgery Test
- Which of the following conditions result in abnormal head turn/tilt?
- Fourth cranial Nerve palsy
- Cervical dystonia
- Ocular tilt reaction
- All of the above
Answer: D
Discussion: The skull houses the head and hence abnormalities of the head are common and often very important in clinical neurological examination. Abnormal turn of head or tilt is indicative of cervical dystonia, a fourth nerve palsy or an ocular tilt reaction. Other developmental anomalies or deformities include hydrocephaly, macro/microcephaly, contour asymmetry, disproportion between facial and cerebral components. Evidence of recent trauma like echymosis around mastoid-Battle's sign, echymosis around rims of the eyes-Raccoon's sign etc point towards base of skull fractures.
Reference: Dejong's Neurological Examination: 8th Edition (South East Asian edition)
- Which of the following is not found in the parietal lobe?
- Precuneus
- Post central gyrus
- Superior parietal lobule
- Precentral gyrus
Answer: D
Discussion: the Parietal lobe lies posterior to the central sulcus, anterior to the occipital lobe and superior to temporal lobe. The parietal lobe consist of 5 principal areas : The post central gyrus, Superior parietal lobule, Inferior parietal lobule, the precuneus and posterior portion of the paracentral lobule. The post central gyrus (areas 1,2 and 3 ) is the primary sensory cortex. The secondary somatosensory cortex lies in the inferior portion of the postcentral gyrus, abutting the sylvian fissure. The superior parietal lobule is a somatosensory association area that lies posterior to the trunk and upper-extremity segments of the postcentral gyrus. The inferior parietal lobule lies posterior to the face and tongue segments of the postcentral gyrus, and it has the following two major components: the supramarginal gyrus, which caps the upturned end of the sylvian fissure; and the angular gyrus, which is at the end of the parallel superior temporal sulcus. The inferior parietal lobule is association cortex for somatosensory, visual, and auditory functions. The precuneus is an area of the cortex just anterior to the occipital lobe on the medial hemispheric surface.
Reference: Snells Clinical Neuroanatomy 8th Edition.
- Which of the following layers in the precentral gyrus contains giant pyramidal cells of Betz?
- Layer I
- Layer III
- Layer V
- Layer VI
Answer: C
Discussion: Most of the cortical mantle has six identifiable layers. Six-layered cortex is referred to as neocortex, isocortex, or heterogenetic cortex. The six layers, from superficial to deep, are as follows: (1) molecular (plexiform), (2) external granular, (3) external pyramidal, (4) internal granular, (5) internal pyramidal (ganglion), and (6) multiform.
Layer 5, the internal pyramidal (ganglion cell) layer, consists of medium and large pyramidal cells, among which are the largest neurons found in the cortex. In the precentral gyrus, this layer contains the giant pyramidal cells of Betz, the neurons whose axons form the corticospinal and corticobulbar tracts.
Reference: Guyton's textbook of medical Physiology 13th edition.
- Which of the following is a disconnection syndrome?
- Conduction aphasia
- Alexia without agraphia
- Pure word deafness
- All of the above
Answer: D
Discussion: Disconnection syndromes are disorders in which the fiber tracts that interconnect primary cortical areas are disrupted, with preservation of the cortical areas of origin. Neurologic dysfunction occurs not because of destruction of cortex but because of defects in intrahemispheric or interhemispheric communication. Condution aphasia was the first disconnection sydrome as explained by Wernicke in 1874. Dejerine explained alexia without agraphia in 1892. Other disconnection syndromes include ideomotor apraxia, sympathetic apraxia, pure word deafness and the transcortical aphasias. Subcortical white matter disruptions including infarction, hemorrhage, neoplasm, and trauma all can cause disconnection syndrome.
Reference: Localization in clinical neurology by Paul Brazis- 7th Edition
- Inability to carry out at request an act in the absence of any weakness, sensory loss, or any deficit related to the affected part is called:
- Agnosia
- Apraxia
- Allodynia
- Prosopagnosia
Answer: B
Discussion: Apraxia is defined commonly as the inability to carry out on request a motor act in the absence of any weakness, sensory loss, or other deficit involving the affected part. The patient must have intact comprehension and be cooperative and attentive to the task. It can vary with what it takes to perform a high-level, learned, familiar, and purposeful task such as saluting or an elemental task like opening or closing the eyes (eyelid apraxia), glancing to the side (ocular motor or gaze apraxia), walking (gait apraxia), or a behavior as basic as smacking the lips (buccofacial apraxia). It can also be an inability to perform an act on command that the patient is able to perform spontaneously. Thus the patient who has gait apraxia cannot walk spontaneously any better than to command. There are many varieties of apraxia. The ones seen most often are ideomotor, buccofacial, constructional, and dressing apraxia.
Reference: Dejong's Neurological Examination: 8th Edition (South East Asian edition)
- Maxillary division of Trigeminal nerve exits the skull through Foramen magnum
- Foramen rotundum
- Superior orbital fissure
- Foramen spinosu
- Answer: B
Discussion: The important foraminae of skull base and their contents as below:
- In conductive hearing loss
- Rinne's test BC>AC (negative) and in Weber's the sound lateralized to abnormal side
- Rinne's test BC>AC (negative) and in Weber's the sound lateralized to normal side
- Rinne's test AC>BC (positive) and in Weber's the sound lateralized to abnormal side
- Rinne's test AC>BC (positive) and in Weber's the sound lateralized to normal side
Answer: A
Discussion: Normally the auditory acuity is equal in both ears, air conduction is greater than bone conduction (Rinne test normal or positive) bilaterally, and the Weber test is nonlateralizing (midline).
The table depicts the pattern on the involved side with unilateral conductive or sensorineural hearing loss.
Auditory Acuity | Rinne Test | Weber Test | |
Conductive hearing loss | Decreased | BC>AC (Rinne negative or abnormal) | Lateralizes to abnormal side |
Sensorineural hearing loss | Decreased | AC>BC (Rinne positive or normal) | Lateralizes to normal side |
Reference: Handbook of Neurosurgery-Greenberg 9th edition
- Gradenigo's syndrome false is:
- V1 sensory loss (first division of the CN V)
- Facial pain
- Mono ocular blindness
- CN VI nerve palsy
Answer: C
Discussion: Gradenigo's syndrome involves Sixth cranial nerve palsy, facial pain and V1 sensory loss sue to lesions at the petrous apex.
One of the most common condition causing Gradenigo's syndrome was suppurative Otitis media spreading to petrous apex of temporal bone. GS may progress from untreated otitis media when the infection spreads to the petrous apex of the temporal bone. Infection is commonly caused by Streptococcus pneumoniae, Haemophilus influenzae, Pseudomonas, and Staphylococcus aureus. Tuberculosis and fungal infection is another less likely cause of infection. Bacteria travels from the middle ear to the mastoid air cells, which contain highly vascular marrow and are susceptible to infection. It can then spread to the adjacent petrous temporal bone, around which are located many key structures. These structures include the trigeminal ganglion and abducens nerve, which are separated from the petrous bone only by dura mater. Inflammation in this region can damage these nerves, causing facial pain and horizontal diplopia secondary to unilateral esotropia.
In this era is very powerful antibiotics and early diagnostic skills the complete form of this syndrome is very rare.
Reference: Neurosurgery Examination Review - Q and A
- Which of the following is a component of Nervus intermedius?
- Pure motor
- Pure sensory
- Motor and sensory
- Sensory and autonomic
Answer: D
Discussion: The nervus intermedius is the sensory and parasympathetic division of the facial nerve. It runs in between 7th and 8th cranial nerves across the CP angle. It contains visceral afferent fibers coming from the taste buds of the anterior two-thirds of the tongue through Corda tympani and mucous membranes of the pharynx, nose, and palate. Sensory stimuli from the skin around the external auditory meatus, lateral pinna, and mastoid region are also conveyed via these afferents. Autonomic components consist of preganglionic general visceral efferent parasympathetic fibres from the superior salivatory and lacrimal nucleus. They travel through the chorda tympani, arriving at their bipolar cell bodies in the geniculate ganglion, and project into the brainstem to the solitary tract nucleus.
Reference: Snells Clinical Neuroanatomy 8th Edition.
- Spasticity or increase in tone due to pyramidal lesions is most marked in:
- Extensors of the Upper limb
- Supinators of the Upper limb
- Extensors of the Lower limb
- Flexors of the lower limb
Answer: C
Discussion: The spasticity is most marked in the flexor and pronator muscles of the upper limb and extensors of the lower limb.
Reference: Dejong's Neurological Examination: 8th Edition (South East Asian edition)
- The organ of Corti responsible for hearing is located on the
- Tympanic membrane
- Vestibular (Reissner's) membrane
- Basilar membrane
- Superior olivary nudeus
Answer: C
Discussion: The organ of Corti is composed of hair cells responsible for hearing. These are located on the basilar membrane that separates scala tympani from scala media and their cilia project into the tectorial membrane. Activation of the hair cells stimulates the cochlear nerve endings. The vestibular (Reissner's) membrane separates scala media from scala vestibuli.
Reference: Guyton's textbook of medical Physiology 13th edition.
- The neuroleptic effects of haloperidol are caused by
- Blocking histamine receptors
- Blocking serotonin receptors
- Blocking a-adrenergic receptors
- Blocking dopamine receptors
Answer: D
Discussion: Neuroleptic effect of Haloperidol is through blocking of dopamine receptors and it rebalances dopamine to improve mood, thinking and behaviour. Haloperidol is a powerful neuroleptic (antipsychotic), used to treat schizophrenia, by blocking dopamine receptors in the limbic system. Side effects are caused by blocking histaminergic, serotonergic, and a-adrenergic receptors.
Reference: Goodman and Gillman Pharmacological basis of therapeutics
- Which of the following cells are an important component of the blood- brain barrier?
- Astrocytes
- Oligodendrocytes
- Microglia
- Neurons
Answer: A
Discussion: Astrocytes are one of the most important glial cells with prominent role in maintaining hemostasis in CNS. The main components of the blood-brain barrier (BBB) are vascular endothelial cells with their tight junctions and basement membranes as well as astrocytic foot processes. By selectively permitting substances into the CNS, BBB protects the brain from various chemicals and toxins. Most circumventricular organs are deficient in blood-brain barrier: pineal gland, median eminence and posterior pituitary, area postrema (paired), organum vasculosum of the lamina terminalis, and subforniceal organ. The only circumventricular organ with intact BBB is the subcommissural organ.
Reference: Robbins and Cotran pathologic basis of Diseases: SE edition
- The anterior neuropore closes at which gestational age?
- 17 days
- 22 days
- 24 days
- 28 days
Answer: C
Discussion: During primary neurulation, the neural tube closure starts in the middle at 22 days, then the anterior neuropore at 24 days (lamina terminalis), and then the posterior neuropore at 28 days.
Reference: Snells Clinical Neuroanatomy 8th Edition.
- All of the following is true about the one-and-a-half syndrome, except
- One eye can't abduct or adduct
- The other eye can't abduct
- There Is bilateral medial longitudinal fasciculus (MLF) injury
- Could be caused by demyelination (commonest being multiple sclerosis)
Answer: B
Discussion: The other eye cant adduct. One-and-a-half syndrome is characterized by bilateral internuclear ophthalmoplegia resulting in bilateral failure of adduction, as well as VI nerve palsy on one side resulting in failure of abduction of the ipsilateral eye. It is caused by bilateral lesion of medial longitudinal fasciculus, plus unilateral VI nerve palsy. Common causes include brain stem ischemia or demyelination.
Reference: Handbook of Neurosurgery-Greenberg 9th edition
- Benedikt's syndrome is caused by a lesion of
- Cranial nerve IV
- Cranial nerve VI
- Red nucleus
- Substantia nigra
Answer: C
Discussion: Benedikt's syndrome is one of the midbrain syndromes commonly due to posterior circulation stroke. Othe causes can be Infarction, tumour or chronic granulomatous infections in the tegmentum of midbrain and may also involve superior cerebellat peduncle. Posterior cerebral artery or rarely penetrating branches of basilar artery occlusion are implicated. The syndrome involves CN III n and red nucleus causing ipsilateral thrid cranila nerve palsy with extra ocular movement disturbances, pupillary defects and contralateral hemi ataxia.
Weber's syndrome is similar but involves ipsilateral Illrd n palsy and contralateral hemiplegia (corticospinal tract)
Reference: Localization in clinical neurology by Paul Brazis- 7th Edition
- The most common location for a saccular brain aneurysm is
- Anterior communicating a (ACOM)
- Posterior communicating a (PCOM)
- Middle cerebral a (MCA)
- Posterior inferior cerebellar a (PICA)
Answer: A
Discussion: Incidence of saccular aneurysms of the brain as below:
- ACOM:40%
- PCOM:30%
- MCA: 20%
- Basilar: 8%
- PICA: 2%
Reference: Youman and Winn Neurological Surgery, 7th Edition
- After a lymph node biopsy of the neck. a patient experiences inability to raise the arm above the head and pain. His wife noticed a winged scapula. What is the most likely injured nerve?
- Long thoracic n
- Spinal accessory n
- Axillary n
- Suprascapular n
Answer: B
Discussion: Injury to the spinal accessory n (cranial nerve XI) is typically seen with lymph node biopsies in the posterior triangle of the neck. The nerve is very superficial in this location. This results in inability to abduct the shoulder beyond 90°, scapular winging, and pain. Suprascapular and axillary nerves do not cause scapular winging. Long thoracic n can give scapular winging and loss of shoulder abduction but the nerve is too deep within the middle scalene muscle and is unlikely to be injured with a lymph node biopsy.
Reference: Neurosurgery Examination Review - Q and A
- The resting membrane potential is maintained by the
- Sodium-potassium pump
- Rapid influx of sodium
- Calcium influx
- Potassium efflux
Answer: A
Discussion: The resting membrane potential is maintained by the sodium-potassium pump;
using 1 ATP (adenosine triphosphate), 3 sodium ions go out causes depolarization, potassium efflux causes repolarization, while calcium influx causes neurotransmitter release from presynaptic terminals.
Reference: Guyton's textbook of medical Physiology 13th edition.
- Sacral sparing in the setting of a spinal cord injury
- Is a good prognostic sign
- Should be retested after return of the bulbo-cavernosus reflex
- Is positive if there is preserved anal sphincter function
- All of the above
Answer: D
Discussion: Sacral sparing is a sign of incomplete spinal cord injury. Return of the bulbocavemosus reflex means that the patient is not in spinal shock (transient loss of function and all reflexes below the level of the injury) anymore, which makes clinical evaluation more accurate. Testing for sacral sparing includes perianal sensation to pin prick, rectal tone, and plantar flexion of the big toe. It is a good prognostic sign due to the proximity of the spinothalamic tract to the lateral corticospinal tract. In both tracts, the sacral fibers are lateral and cervical fibers medial as topographically arranged.
Bulbocavemosus reflex is the first reflex to return once the patient recovers from spinal shock.
Spinal shock is not the same as neurogenic shock. Neurogenic shock is a due to sympathetic failure and usually occurs in lesions above T6 and is associated with hemodynamic collapse.
Reference: Handbook of Neurosurgery-Greenberg 9th edition
SURGICAL ONCOLOGY Test
- Which of the following statements is correct regarding hedgehog signalling pathway mutations in Basal cell carcinoma?
- Activating mutations in SMO (Smoothened) and inactivating mutations in PTCH(Patched)
- Inactivating mutations in SMO (Smoothened) and activating mutations in PTCH(Patched)
- Activating mutations in both SMO and PTCH
- Inactivating mutations in both SMO and PTCH
Answer – A; Activating mutations in SMO(Smoothened) and inactivating mutations in PTCH(Patched) ( sabiston first SA ed; pg 748)
On a molecular level, 90% of BCCs have been linked to mutations in the hedgehog signaling pathway. A key signaling pathway in embryonic development, it is largely inactive in adult tissues. In the presence of hedgehog (Hh) signaling peptides, the Patched (PTCH) receptor releases the transmembrane Smoothened (SMO) protein, allowing SMO to initiate a signaling cascade that activates the expression of several target genes. Normally, PTCH will inhibit SMO in the absence of Hh signals. Both activating mutations in SMO and inactivating mutations in PTCH have been linked to BCC, ultimately leading to unrestricted growth signalling.
- The recommended margin for wide local excision of melanoma involving trunk with 3mm thickness is ....
- 1cm
- 2 cm
- 5 cm
- 3 cm
Answer – B; 2cm ( sabiston first SA ed; pg 734)
Tumor thickness | Recommended surgical margin |
In Situ | 0.5 – 1 cm |
<1mm | 1cm |
1 -2 mm | 1-2 cm |
>2 – 4 mm | 2 cm |
>4mm | 2 cm |
- A 45 years old never smoker male was diagnosed with adenocarcinoma of the lung one year back. Molecular testing revealed an exon 19 deletion in EGFR. He was started on EGFR inhibitor erlotinib and had a partial response. However his recent scan revealed significant progression of disease with multiple new lesions in liver and lung. His tumor is rebiopsied. Which of the following alterations is most likely to explain the tumor’s resistance to erlotinib
- L858R
- MET amplification
- T790M
- The biopsy is likely to reveal transdifferentiation to small cell lung carcinoma
Answer : C ; T790M ( casciato’s manual of clinical oncology -8th ed ; pg 187 )
The activity of EGFR-TKIs differs among various types of EGFR mutations with deletion mutations in exon 19 responding more favourably than exon 21 L858R mutations. The eventual development of acquired resistance generally limits the duration of response to EGFR-TKIs. Almost 50% of patients whose disease progresses on an EGFR-TKIs develop an EGFR T790M gatekeeper mutation on exon 20. Less common resistance mechanisms include MET amplification, PIK3CA mutation, epithelial to mesenchymal transition and SCLC transformation. Osimertinib has been approved for treatment of patients with tumors harboring acquired T790M mutations
- All the following statements are true about MYH –associated polyposis except...
- Phenotypically it is indistinguishable from attenuated Familial Adenomatous polypopis
- Autosomal dominant inheritance
- MYH gene encodes a DNA glycosylase involved in base excision repair pathway
- Extracolonic manifestations include breast cancer and upper gastrointestinal polyps
Answer : B; Autosomal dominant inheritance ( sabiston first SA ed; pg 692)
MYH-associated polyposis (MAP) is a syndrome caused by mutations in the human MutY homologue (MYH) gene. It accounts for about a third of patients who have attenuated polyposis but who test negative for APC mutations. Unlike FAP, MAP is inherited in an autosomal recessive manner. Phenotypically, MAP-associated colorectal cancer is indistinguishable from attenuated FAP, although it is manifested later, around the age of 50 years. The polyps are distributed throughout the colon, although there are conflicting data about right- and left-sided tumor predominance.
Extracolonic manifestations include breast cancer (18%) and upper gastrointestinal polyps (one third). The MYH gene encodes a DNA glycosylase involved in the base excision repair pathway, important in preventing mutations due to oxidative damage. Y165C and G382D mutations account for more than 80% of all mutations discovered thus far. Penetrance is estimated at 50%. Homozygotes or compound heterozygotes for germline mutations of the MYH gene have a 93-fold increased risk of colorectal cancer. Mutation leads to chromosomal instability in which there is an accelerated rate of chromosomal misaggregation during cell division. This leads to aneuploidy, which has been recognized as an early genetic change in the stepwise carcinogenesis of both FAP and MAP tumors. Polyps bearing MYH mutations have twice the overall incidence of aneuploidy compared with those in patients with FAP. Current evidence suggests that carriers of single mutated alleles are unlikely to have more than a 50% increased risk of colorectal cancer.
- All of the following are paraneoplastic syndromes associated with thymoma except..
- Myasthenia gravis
- Pure red cell aplasia
- Good syndrome
- Eaton-Lambert myasthenic syndrome
Answer : D; Eaton-Lambert myasthenic syndrome ( casciato’s manual of clinical oncology -8th ed ; pg 446 )
PNS associated with thymoma
- Myasthenia gravis(>50%)
- Pure red cell aplasia
- Immunodeficiency – acquired hypogammaglobinemia with low to absent B cells & CD4+ T lymphocytopenia ( GOOD SYNDROME)
- Rare PNS – ectopic cushing syndrome, polymyositis, dermatomyositis, granulomatous myocarditis, SLE, churg straus syndrome, microscopic polyangitis,isolated pauci-immune necrotizing crescentric glomerulonephritis,optic neuritis, limbic encephalitis, hypertrophic osteoarthropathy, TAMA(thymoma associated multiorgan autoimmunity
- Which of the following is false about Oncotype Dx breast cancer assay for predicting breast cancer recurrence..?
- 21 gene recurrence score assay
- Validated first in patients with stage I or II, node negative, ER positive breast cancer
- The score ranges between 0 to 50
- Score of 31 or more indicates a high risk group
Answer : C; The score ranges between 0 to 50 (sabiston first SA ed; pg 855; casciato’s manual of clinical oncology -8th ed ; pg 277)
Multigene assays, such as the 21-gene recurrence score assay (Oncotype DX Breast Cancer Assay, Genomic Health, Inc., Redwood City, CA), have been developed in an attempt to identify a specific molecular phenotype of a tumor in an individual patient and use the phenotype to predict the response to therapy or provide information regarding prognosis. The Oncotype DX assay was developed from a candidate pool of 250 genes and narrowed to a specific 21-gene panel on the basis of three independent studies of the candidate genes. This assay was validated first in patients with ER-positive, lymph node–negative breast cancer (NSABP B-14). The Oncotype DX assay was found to be prognostic for OS and predictive of the benefits of different systemic therapies, with higher recurrence scores predicting increased benefit from chemotherapy and lower scores predicting lesser benefit from chemotherapy and increased benefit from endocrine therapy. This assay was validated in subsequent studies.
The Oncotype DX assay can help clinicians estimate the benefits of therapy for patients with lymph node–negative, ER-positive breast cancer. For patients with low recurrence scores, chemotherapy appears to have marginal benefit in terms of reducing the risk of distant recurrence, but for patients with high recurrence scores, chemotherapy offers marked benefit. For patients with intermediate recurrence scores, the magnitude of benefit from chemotherapy is uncertain.
- According to Haggits classification for polyps containing cancer according to depth of invasion, which level is the carcinoma invading into the submucosa of the bowel wall below the stalk of the polyp but above the muscularis propria?
- Level 1
- Level 2
- Level 3
- Level 4
Answer : D; level 4 (sabiston first SA ed; pg 1365)
Haggitt and colleagues have proposed a classification for polyps containing cancer according to the depth of invasion, as follows
Level 0: Carcinoma does not invade the muscularis mucosae (carcinoma in situ or intramucosal carcinoma)
Level 1: Carcinoma invades through the muscularis mucosae into the submucosa but is limited to the head of the polyp
Level 2: Carcinoma invades the level of the neck of the polyp (junction between the head and stalk)
Level 3: Carcinoma invades any part of the stalk
Level 4: Carcinoma invades into the submucosa of the bowel wall below the stalk of the polyp but above the muscularis propria
All sessile polyps with invasive carcinoma are level 4 by haggits criteria
- A 40 years old female presenting with 3cm lump in left breast with ipsilateral satellite nodules and pectoralis muscle invasion with ipsilateral matted axillary nodes and no evidence of distant metastasis..AJCC staging is..
- T4b N2a M0 (stage IIIB)
- T4c N2a M0 (stage IIIB)
- T4b N2b M0 (stage IIIB)
- T4b N3a M0 (stage IIIC)
Answer : A; T4b N2a M0 (stage IIIB)
T staging
TX -Primary tumor cannot be assessed
TO- No evidence o f primary tumor
Tis (DCIS)*- Ductal carcinoma in situ
Tis (Paget) -Paget disease o f the nipple NOT associated with invasive carcinoma and/or carcinoma in situ (DCIS) in the underlying breast parenchyma. Carcinomas in the breast parenchyma associated with Paget disease are categorized based on the size and characteristics of the parenchymal disease, although the presence of Paget disease should still be noted.
TI - Tumor < 20 mm in greatest dimension
T1 mi - Tumor < 1 mm in greatest dimension
T1a - > 1 mm but < 5 mm in greatest dimension
(round any measurement 1.0-1.9 mm to 2 mm).
T1b - Tumor > 5 mm but < 1 0 mm in greatest dimension
T1c - Tumor > 10 mm but < 20 mm in greatest dimension
T2 - Tumor > 20 mm but < 50 mm in greatest dimension
T3 - Tumor > 50 mm in greatest dimension
T4 - Tumor o f any size with direct extension to the chest wall and/or to the skin (ulceration or macroscopic nodules); invasion o f the dermis alone does not qualify as T4
T4a- Extension to the chestwall; invasion or adherence to pectoralis muscle in the absence of invasion of chest wall structures does not qualify as T4.
T4b- Ulceration and/or ipsilateral macroscopic satellite nodules and/or edema (including peau d ’orange) of the skin that does not meet the criteria for inflammatory carcinoma
T4c- Both T4a and T4b are present
T4d - Inflammatory carcinoma
* Note: Lobular carcinoma in situ (LCIS) is a benign entity and is removed
from TNM staging in the AJCC Cancer Staging Manual, 8lh Edition.
N staging
NX-Regional lymph nodes cannot be assessed (e.g.,previously removed)
N0-No regional lymph node metastases (by imaging or clinical examination)
N1 -Metastases to movable ipsilateral Level I. II axillarylymph node(s)
N1mi- Micrometastases (approximately 200 cells, larger than 0.2 mm, but none larger than 2.0 mm)
N2- Metastases in ipsilateral Level I. II axillary lymph nodes that are clinically fixed or matted; or in ipsilateral internal mammary nodes in the absence o f axillary lymph node metastases
N2a- Metastases in ipsilateral Level I, II axillary lymph nodes fixed to one another (matted) or to other structures
N2b- Metastases only in ipsilateral internal mammary nodes in the absence o f axillary lymph node metastases
N3 - Metastases in ipsilateral infraclavicular (Level III axillary) lymph node(s) with or without Level 1. II axillary lymph node involvement; or in ipsilateral internal mammary lymph node(s) with Level I, II axillary lymph node metastases; or m etastases in ipsilateral supraclavicular lymph node(s) with or without axillary or internal mammary lymph node involvement
N3a - Metastases in ipsilateral infraclavicular lymphnode(s)
N3b - Metastases in ipsilateral internal mammary lymph node(s) and axillary lymph node(s)
N3c - Metastases in ipsilateral supraclavicular lymph node(s)
- A 65 years old male underwent Right lower lobectomy and systematic lymph node sampling for a stage I adenocarcinoma. The lymph nodes are minimum sampled from the all the following stations except...
- 6
- 7
- 8
- 9
Answer : A; 6 (sabiston first SA ed; pg 1590)
Stage I and II NSCLC can be treated safely with surgery and mediastinal lymph node dissection alone, and most patients have long-term survival. Anatomic resection with lobectomy, with systematic mediastinal lymph node dissection/sampling, is the procedure of choice for lung cancer confined to one lobe . The American College of Surgeons Oncology Group defined a systematic sampling strategy for specific mediastinal lymph nodes.
At a minimum, samples of nodal (not adipose) tissue from stations 2R, 4R, 7, 8, and 9 for right-sided cancers and stations 4L, 5, 6, 7, 8, and 9 for left-sided cancers should be obtained. Mediastinal lymphadenectomy should include exploration and removal of lymph nodes from stations 2R, 4R, 7, 8, and 9 for right-sided cancers and stations 4L, 5, 6, 7, 8, and 9 for left-sided cancers.
- Type II cholangiocarcinoma which requires a partial hepatic resection which commonly includes segment...
- I
- II
- III
- IV
Answer : A; I
Surgical management of proximal cholangiocarcinoma involves resection of regional nodal tissue and en bloc resection of the common bile duct with hepatic parenchyma as necessary to achieve negative margins. The Bismuth-Corlette classification of the tumor by assessment of the involvement of biliary radicals helps with operative planning. Types I and II lesions are treated with common duct resection, cholecystectomy, and a 5- to 10-mm margin of resection. Type II lesions may also require partial hepatic resection, which commonly includes resection of the caudate lobe. Resection of the bile duct and nodal tissue requires skeletonization of the hepatic artery and portal vein. Reconstruction is performed using a Roux limb of jejunum. Types III and IV lesions may involve complex resection and reconstruction of the portal vein, hepatic artery, or both. With resection to secondary biliary radicals, transanastomotic stenting is used liberally to allow healing and even confirmation of anastomotic integrity.
- Which genetic change in colorectal cancer is associated with resistance to treatment with cetuximab?
- KRAS mutation
- EGFR overexpression
- KRAS wildtype
- MLH 1 inactivation
Answer : A; KRAS mutation (sabiston first SA ed; pg 1377 ; Bethesda handbook of clinical oncology 5th ed, pg 113)
Cetuximab is a chimeric (mouse-human) monoclonal antibody; panitumumab, a fully human monoclonal antibody, binds to and inhibits the EGFR, which is overexpressed in 60% to 80% of colorectal cancers and is associated with a shorter survival time. Cetuximab and panitumumab are effective only on tumors that do not have a mutation of the KRAS gene. Accordingly, genetic testing is now recommended to confirm the absence of KRAS mutations (indicating the presence of the KRAS wild-type gene) before the use of these EGFR inhibitors is recommended. These agents have shown clinical efficacy in patients with metastatic colorectal cancer, both as monotherapy and in combination with irinotecan and FOLFOX. Bevacizumab, a vascular endothelial growth factor inhibitor, has also improved survival when added to regimens that include irinotecan, 5-FU– leucovorin, or oxaliplatin.
- Anal canal carcinoma with involvement of inguinal and external iliac lymph nodes. Nodal staging according to AJCC 8th edition is...
- N1a
- N1b
- N1c
- M1
Answer : C ; N1c (AJCC 8th edition)
NX – Regional lymph nodes cannot be assessed
N0 – No regional Lymph node metastasis
N1- Metastatsis in inguinal, mesorectal, internal iliac or external iliac nodes
N1a – Inguinal, mesorectal or internal iliac lymph nodes
N1b – external iliac lymph nodes
N1c – Metastasis in external iliac with any N1a nodes
- All the following findings indicate a poor prognosis in Nonseminomatous germ cell tumor except....
- Mediastinal primary
- Non pulmonary visceral metastasis
- AFP > 10000ng/ml or HCG >50000 IU/L
- Retroperitoneal primary
Answer : D ; Retroperitoneal primary (Bethesda handbook of clinical oncology 5th ed; pg 221; devita 11 th ed)
Good prognosis:
Testis/retroperitoneal primary. No non pulmonary visceral metastasis. AFP<1000ng/ml; HCG<5000 IU/L;LDH <1.5 x ULN
Intermediate prognosi Testis/retroperitoneal primary. No non pulmonary visceral metastasis. AFP -1000 to 10000ng/ml; HCG - 5000 to 50000 IU/L;LDH - 1.5 to 10 x ULN
Poor prognosis
Mediastinal primary or non pulmonary visceral metastasis. AFP >10000ng/ml; HCG - >50000 IU/L;LDH >10 x ULN
- CT scan of a 65 years old female revealed a 4x3 cm cystic mass in the head of the pancreas showing central calcification with radiating septa giving sunburst appearance. Analysis of cyst fluid showed low amylase and low CEA. The most probable diagnosis is...
- Serous cystic Neoplasm
- Mucinous cystic neoplasm
- IPMN
- Pancreatic adenocarcinoma
Answer – A; Serous cystic Neoplasm (sabiston first SA ed ;pg 1538)
Compared with MCNs, serous cystic neoplasms (SCNs) have a predilection for the head of the pancreas and occur in patients with a higher median age. Patients commonly present with vague abdominal pain and less frequently with weight loss and obstructive jaundice. On gross inspection, SCNs are large, well-circumscribed masses. Microscopic examination reveals multiloculated, glycogen-rich small cysts. Central calcification, with radiating septa giving the sunburst appearance, is a radiographic sign on CT in 10% to 20% of patients
- In grouping of regional lymph nodes in carcinoma stomach, lymph nodes in splenic hilum is...
- Station 9
- Station 10
- Station 11
- Station 12
Answer : B ; Station 10 (sabiston first SA ed; pg 1219)
Lymph node station | Description |
1 | Right paracardial |
2 | Left paracardial |
3 | Lesser curvature |
4sa | Short gastric |
4sb | Left gastroepiploic |
4d | Right gastroepiploic |
5 | Suprapyloric |
6 | Infrapyloric |
7 | Left gastric artery |
8a | Anterior common hepatic |
8p | Posterior common hepatic |
9 | Celiac artery |
10 | Splenic hilum |
11p | Proximal splenic |
11d | Distal splenic |
12a | Left hepatoduodenal |
12b,p | Posterior hepatduodenal |
13 | Retropancreatic |
14v | Superior mesenteric vein |
14a | Superior mesenteric artery |
15 | Middle colic |
16al | Aortic hiatus |
16a2,b1 | Paraaortic middle |
16b2 | Para-aortic, caudal |
- All the following statements are true regarding dermatofibrosarcoma protuberans except...
- t(17,22) is seen in more than 90% patients
- Trunk is the most common site
- It is high grade sarcoma arising from dermal fibroblasts
- Wide local excision with 2 to 4 cm margin is treatment of choice
Answer : C ; It is high grade sarcoma arising from dermal fibroblasts (sabiston first SA ed; pg 750)
Dermatofibrosarcoma protuberans (DFSP) is a low-grade sarcoma arising from dermal fibroblasts. Lesions appear as smooth, flesh-colored nodules in or immediately beneath the skin and generally are manifested in patients between 20 and 50 years of age. Most appear on the trunk (50%), with the remainder on the proximal extremities (20% to 35%) or on the head and neck (10% to 15%). Because of their slow growth, lesions are commonly 1 to 5 cm at diagnosis. Their external appearance belies their true character as tumor cells will frequently invade the underlying soft tissues and contribute to incomplete excision and local recurrence. Treatment consists of WLE with a 2- to 4-cm margin. Specimen orientation and pathologic analysis of margins are required. Distant metastases are uncommon and are preceded by two or more local recurrences. A variant of DFSP is associated with fibrosarcomatous change on pathologic examination; these lesions may have a more aggressive course with a higher risk for distant disease. Adjuvant radiation therapy has been used effectively after resection for recurrences. Interestingly, the translocation t(17:22) has been found in approximately 90% of DFSP lesions. This has led to the use of imatinib in patients with locally advanced or metastatic disease and sometimes as neoadjuvant therapy.
- Which of the following is true about type 2 endometrial carcinomas?
- Endometroid histology
- Often associated with diabetes mellitus and obesity
- Younger patients
- Serous and clear cell histology
Answer : D; Serous and clear cell histology (devita 11th ed and Bethesda handbook of clinical oncology 5th ed; pg 247)
Type 1 endometrial Ca
- Endometroid histology
- More differentiated
- Less myometrial invasion
- Younger patients
- Often associated with DM & obesity
- Better prognosis
- Estrogen dependent
- Mutations in kras,B catenin,PI3K,PTEN,ARID1A, microsatellite instability
Type 2 endometrial Ca
- Non endometroid histology (serous and clear cell)
- P53 mutations,chromatin remodelling and ubiquitin ligase complex genes
- Aneuploid (grade 3)
- Her2/neu overexpressed
- Poorly differentiated
- Older, thin post menopausal women
- Which of the following differentiated carcinoma of thyroid tends to occur in older individuals consisting of oxyphilic cells and have a greater likelihood of having local recurrence and are less avid to absorb RAI leading to more aggressive biological behaviour...?
- Hurthle cell carcinoma
- Classical Papillary carcinoma
- Follicular carcinoma
- Follicular variant of papillary carcinoma
Answer : A; hurthle cell carcinoma (sabiston first SA ed; pg 904-905)
A subtype of FTC, known as Hürthle cell carcinoma, consists of oxyphilic cells and tends to occur in older patients, usually 60 to 75 years old. Hürthle cells have their characteristic appearance because of an increased number of mitochondria causing an appearance of an enlarged, granular, and eosinophilic cytoplasm. Hürthle cell cancers have a greater likelihood of having local recurrence and are less avid to absorb RAI, leading to more aggressive biologic behavior.
- The Seatle biopsy protocol is done for mapping....
- Adenomatous colonic polyps
- Barret oesophagus
- Peptic ulcer disease
- Carcinoma stomach
Answer : B; Barret esophagus (sabiston first SA ed; pg 1033)
The Seattle biopsy protocol is still widely accepted for mapping of Barrett esophagus with high-grade dysplasia. This involves four-quadrant biopsies at 1-cm intervals along the entire length of Barrett esophagus in addition to targeted biopsies of all visible lesions.
- In kindreds with MEN2B RET mutation,Prophylactic total thyroidectomy should be done within...
- 1 year
- 2 years
- 4 years
- 5 years
Answer : A; 1 year (sabiston first SA ed; pg 909)
Patients with inherited MTC syndromes initially develop C cell hyperplasia, which is a preneoplastic lesion in these patients, although C cell hyperplasia has little to no malignant potential in patients without RET mutations. Because of the high penetrance of MTC and the early development of C cell hyperplasia and MTC, family members of patients with MEN2 should be screened at an early age for the RET proto-oncogene. RET testing should be performed shortly after birth in MEN2B kindreds and before age 5 years in FMTC and MEN2A kindreds.
Patients with the MEN2B RET mutation are advised to undergo prophylactic total thyroidectomy within the first year of life or at the time of diagnosis. Other patients with germline RET mutations should undergo prophylactic total thyroidectomy before age 5 years or at the time of diagnosis, although it may be appropriate to wait beyond 5 years with particular RET mutations. Level VI nodal dissection may be omitted in patients younger than 1 year with MEN2B and patients younger than 5 years with MEN2A and FMTC who are undergoing prophylactic thyroidectomy unless there are thyroid nodules larger than 5 mm, elevated calcitonin, or evidence of lymph node metastasis.
Surgical Gastro Test
- Starburst appearance
- MCN
- SCN
- Pseudo cyst
- IPMN
Answer: B (Sabiston 20e, page 1538)
SCN
- Patients commonly present with vague abdominal pain and less frequently with weight loss and obstructive jaundice
- Microscopic examination reveals multiloculated, glycogen-rich small cysts
- Central calcification, with radiating septa giving the sunburst appearance, is a radiographic sign on CT in 10% to 20% of patients
- Differential cyst fluid protein expression was observed between scns and ipmns, with accurate discrimination in 92% of patients
- Select patients with large (>4 cm) or rapidly growing lesions, resection of an SCN is appropriate.
- Type 3 haggit
- Involvement Of Stalk
- Involvement Of Head
- Involvement Of Neck
- Involvement Of Submucosa Of Bowel Wall
Answer: A (Sabiston 20e, page 1365)
Level 0: Carcinoma does not invade the muscularis mucosae (carcinoma in situ or intramucosal carcinoma)
Level 1: Carcinoma invades through the muscularis mucosae into the submucosa but is limited to the head of the polyp
Level 2: Carcinoma invades the level of the neck of the polyp (junction between the head and stalk)
Level 3: Carcinoma invades any part of the stalk
Level 4: Carcinoma invades into the submucosa of the bowel wall below the stalk of the polyp but above the muscularis propria
- Which of the following is not degraded by colonic flora?
- Pectin
- Lignin
- Starch
- Glucose
Answer: B (Sabiston20e, page 1320-1321)
- Probiotic
- Dietary supplement contains live bacteria/yeast
- Mostly:
- Bifidobacterium
- Lactobacillus
- Stimulate immune
- Antiinflammatory
- Decrease pathogenic colonization
- Increase digestion of proten
- Increase absorption of AA
- C defifile-protect/RX
- Treatment of choice for Type 3 cholydochal cyst is ?
- Cyst excision
- 6 monthly followup
- Trans suodenal sphincteroplasty
- Liver transplantation
Answer: C (Sabiston20e, page 1510-1511)
- M<F = 1:8
- 60% < 10 yrs
- ABPJ : 90%
- Triad
- Jaundice (MC)
- RUQpain
- Palpable mass
Type | Treatment |
1 | • Rou en y hepaticojejunostomy
• pericyst fibrosis: intramural plane developed , entire epithelium removed |
2 | Excised
If ABPJ: Biliary enteric anastomosis |
3 | • Endoscopic drainage
• Duodenal /biliary obstruction : transduodenal excision / sphinteroplasty |
4 | • Extrahepatic only : same as type 1
• Both intra/extra: limited unilobar: partial hepatectomy • Bilobar: Transplant |
5 | • limited unilobar: partial hepatectomy
• Bilobar: Transplant |
- Most common complication of hydatid cyst is ?
- Secondary infection
- Rupture into biliary tree
- Rupture into peritoneal cavity
- Rupture into thoracic cavity
Answer: B ( Blumgart 6e, page 1105)
Complication of Hydatid cyst
- Rupture Into the Biliary Tract
- Intrabiliary rupture is often reported as the most common complication of liver hydatid cysts, but its real incidence is difficult to evaluate
- The reported incidence of clinically evident cystobiliary communications rates vary from 1% to 42%
- Some authors state that up to 90% of the liver hydatid cysts have some kind of communication during their evolution
- Predictive factors for cystobiliary communication are
- Age,
- Jaundice,
- Past history of hydatidosis,
- Preoperative level of γ-glutamyl transferase (ggt),
- Pericyst aspect,
- Morphologic cyst aspect, and
- Location near the hilum
- Cyst diameter greater than 10 cm is an independent clinical predictor for the presence of intrabiliary rupture
- diameter > 75 mm : likelihood of cystobiliary fistula is 79%
- When the size of intrabiliary rupture is greater than 5 mm, so-called major biliary communication cyst content is seen in the bile duct in 65% of patients
- The reported incidence of major biliary-cyst communications ranges from 5% to 10%
- Rupture Into the Bronchial Tree
- hydatid cysts of the posterior and upper segments of the liver (IVa, VII, and VIII) may stick to the diaphragm, causing indentation, and ultimately spontaneous erosion
- Rupture Into the Peritoneum
- This complication is rare, even in endemic regions, with an incidence ranging from 1% to 8%
- Systemic anaphylactic reactions have been reported in 1% of the patients with intraperitoneal perforations
- Rupture Into Other Cavities or Organs
- Compression
- Cyst Infection
- SADI-S , false statement
- Stands for single anastomosis duodenoileal bypass with sleeve gastrectomy
- Based on BPD-DS
- Length of common channel alimentary limb 200cm
- Pylorus not preserved
Answer: D (Bailey 27e, page 1150)
- Multichannel intraluminal impedance, false statement?
- Requires an alternating current source
- Liquid boluses conduct less than the empty esophagus, leading to a slow decline in intraluminal impedance
- allow determination of the direction of bolus movement
- Can be combined with Ph manometry
Answer: B (Shakelford 8e, page 136)
- First described by Silny in 1991
- The presence and movement of an intraesophageal bolus is detected on MII based on measuring differences in electrical conductivity affected by the presence of various materials within the esophagus
- An MII catheter requires an alternating current source connected to a series of metal rings located along segments of the esophageal lumen. An isolator separates the rings so that the electrical circuit is closed by the electrical charges (i.e., ions) surrounding the catheter
- The impedance within a given segment is then determined by measuring the electrical resistance as a substance passes through the current established by the rings.
- Once placed in the esophagus, the ions of the esophageal mucosa close the circuit and the system measures a relatively stable resistance of approximately 2000 to 3000 ohms
- Liquid boluses conduct better than the empty esophagus, leading to a rapid decline in intraluminal impedance when the bolus enters the impedance measuring segment. Impedance returns to baseline once the bolus has exited the segment.
- Multiple impedance measuring segments mounted on the same catheter allow determination of the direction of bolus movement based on the timing of changes in impedance at individual levels. Proximal to distal (antegrade) progression of impedance changes is indicative of swallowing (Fig. 9.6), whereas a distal to proximal (retrograde) progression indicates a reflux episode
- When combined with pH, MII can evaluate the presence of a reflux event and its pH, thereby allowing the detection and differentiation of both acid and non-acid reflux
- Prophylactic Gastrectomy, False statement?
- Once the mutation has been diagnosed, prophylactic gastrectomy is recommended for asymptomatic carriers between the ages of 10-18 yrs
- 1% to 3% of gastric cancers are hereditary in nature, with the most common type being hereditary diffuse gastric cancer
- Screening recommendation: gastric cancer in one patient and breast cancer in one first-degree or second-degree relative with one diagnosis before age 50
- Screening recommendation: gastric cancer in three or more first-degree or second degree relatives regardless of the age of onset,
Answer: A (SHakelford 8e, page 717)
Overall, 1% To 3% Of Gastric Cancers Are Hereditary In Nature, With The Most Common Type Being Hereditary Diffuse Gastric Cancer. This Is Characterized By An Autosoma Ldominant Inheritance Pattern And Diffuse Signet Ring Cells Due To A Germline Mutation In CDH1 (E-Cadherin).
Guidelines For Screening Include:
- A Known Mutation In A Gastric Cancer Susceptibility Gene Within The Family,
- Gastric Cancer In One Family Member Before Age 40,
- Gastric Cancer In Two First-Degree Or Second-Degree Relatives With At Least One Case Diagnosed Before Age 50,
- Gastric Cancer In Three Or More First-Degree Or Second Degree Relatives Regardless Of The Age Of Onset,
- Gastric Cancer And Breast Cancer In One Patient With One Diagnosis Before Age 50, Or
- Gastric Cancer In One Patient And Breast Cancer In One First-Degree Or Second-Degree Relative With One Diagnosis Before Age 50.
Once The Mutation Has Been Diagnosed, Prophylactic Gastrectomy Is Recommended For Asymptomatic Carrier Between The Ages Of 18 And 40. Annual Surveillance Endoscopy With Biopsy Should Be Performed For Those Patients Who Elect Not To Undergo Prophylactic Gastrectomy, Although The Efficacy Of This Practice Is Not Well Established due to the diffuse nature of these malignancies
- Small bowel diverticula, true statement?
- Ileal diverticula more common than Jejunal diverticula
- Hemorrhage is most common presentation
- Gastric tissue is most common type of heterotropic tissue in symptomatic meckels diverticulum
- Jejunal diverticula mostly symptomatic
Answer: C (Shakelford, 8e page 909-911)
- Jejunal and ileal diverticula are the least common small bowel diverticula, with an incidence of 0.2% to 5% based on imaging and autopsy data.
- 6th-7th decade: peak
- 60% to 70% of patients with jejunoileal diverticula are asymptomatic
- who develop chronic symptoms or acute complications, only 10% will require surgery
- false diverticula that penetrate through the mesenteric side of the bowel
- Diverticulitis is the most common presentation, accounting for up to 55% of complications
- Most perforations will be walled off by the surrounding mesentery or small bowel. The presentation and imaging may be suspicious for perforated colonic diverticulitis or appendicitis depending on the location of the abscess. The mortality rate for perforated diverticulitis can reach 50% due to the difficulty, and subsequent delay, in diagnosis
- Hemorrhage from a jejunoileal diverticulum can be seen in 5% to 33% of patients with acute symptomatic disease
- Bowel obstruction due to jejunoileal diverticulosis is the least common complication
- Asymptomatic jejunoileal diverticula should be left alone when discovered incidentally
- However, patients who experience complications related to small bowel diverticular disease should undergo segmental resection with primary anastomosis.
- There is no role for invagination of diverticula or simple diverticulectomy because this results in poor blood flow, ischemia, and a high leak rate
- Meckels diverticulum
- Rule of 2s:
- located 2 feet (60 cm) from the terminal ileum,
- 2 inches in length,
- affecting 2% of the general population,
- occurring twice as often in males,
- containing one or two types of heterotopic mucosa (most commonly gastric or pancreatic), and
- presenting in the first 2 decades of life,
- most commonly in the first 2 years of life
- Meckel diverticulum is by far the most common, representing 90% of vitelline duct anomalies.
- Upto 60% of Meckel diverticula have heterotopic mucosa.
- Although pancreatic tissue is the most common type of heterotopic tissue,
- gastric tissue is the most common type in a symptomatic Meckel diverticulum, as later discussed.
- A Meckel diverticulum is usually positioned within 100 cm of the ileocecal valve, although the mean distance varies with age: the older the patient, the farther away the Meckel diverticulum is from the ileocecal valve.
- < 2 years of age, the mean distance is 34 cm,
- 3 to 21 years: 46 cm
- >21 yrs: 67 cm
- Short bowel syndrome , True statement
- Multiple resection rather than massive resection is more common cause for SBS
- Anatomic criteria for Bianchi LILT : length of residual small bowel greater than 30 cm,
- STEP: placing alternating and opposite transverse staple fires parallel to the mesentery
- Teduglutide is the recombinant human analogue of glucagon-like peptide 1 (GLP-1)
Answer: C (Shakelford 8e, page 920-937)
Approximately 15% of all adults who undergo bowel resection exhibit sequelae of SBS, from either massive resection (76%) or multiple resections (24%)
small intestine has a large functional reserve capacity, and resection of up to 50% of intestinal length can be well tolerated. However, crossing the threshold of less than 200 cm of residual intestine leads to some of the clinical sequelae of SBS in at least 50% of these patients
- Acute phase—postresection to 4 weeks. The goal is stabilization of the patient’s sequelae of diarrhea, malabsorption, and dysmotility.
- Adaptive phase—1 to 2 years. The goal is achieving maximal intestinal adaptation with a gradual increase of nutritional exposure.
- Maintenance phase—long term. Optimizing fluid balance and individualized dietary regimen. Management of acute exacerbations.
LONGITUDINAL INTESTINAL LENGTHENING AND TAILORING (LILT)
In 1980 Bianchi reported the LILT procedure
In 1981 the first clinical application was reported by Boeckman et al., when LILT was used successfully on 50 cm of dilated bowel in a 4-year-old male with gastroschisis and intrauterine bowel necrosis
This anatomy
allows for the two layers of mesentery containing the blood vessels to be separated bluntly on the mesenteric side and bowel divided longitudinally along each parallel lumen
These two fully vascularized small bowel segments are anastomosed isoperistaltically in a gentle S loop, effectively tapering and doubling the length of the original segment
Anatomic criteria for patient selection include
(1) intestinal diameter greater than 3 cm,
(2) length of residual small bowel greater than 40 cm, and
(3) length of dilated bowel greater than 20 cm
IOWA
- The Iowa two-step elongation procedure was developed by Kimura et al. And reported in 1993
- For bowel mesentery that is shortened or when the only short gut remaining is a dilated duodenum, the Iowa procedure might be an alternative to the LILT
- Initial surgery consists of deseromyotomizing the antimesenteric surface of the dilated segment of bowel to a host organ, such as deperitonealized abdominal wall (Iowa model I), decapsulized liver (Iowa model II), and adjacent bowel with incised serosa (Iowa model III)
SERIAL TRANSVERSE ENTEROPLASTY
- In 2003 Kim et al. Published a description of the STEP technique in six pigs.
- The STEP relies on the anatomic premise that the small bowel blood supply from the mesentery runs perpendicular to the long axis of the small bowel. Therefore placing alternating and opposite transverse staple fires parallel to the mesentery along the length of the dilated bowel results in a zig zag–shaped elongated bowel with minimal vascular compromise.
- An average of 10 to 20 staple lines are used, reducing the average dilated diameter of 5 to 6 cm to 2 cm and increasing the ultimate length to 1.5- to 2-fold
SPIRAL INTESTINAL LENGTHENING AND TAILORING (SILT)
- The latest innovation in the bowel lengthening is the spiral intestinal lengthening and tailoring (SILT). In 2013 Cserni et al. Published the description of SILT successfully performed on six Vietnamese minipigs
- Cutting the bowel and its associated mesentery along a 45- to 60-degree spiral line, then longitudinally tightening the spiral and lengthening the bowel
- X marks the spot sign seen in:
- Sigmoid volvulus
- Gall stones
- Hepatic hemangioma
- SCN
Answer: A (Shakelford 8e, page 1809)
Sigmoid Volvulus
In anatomic studies, Bhatnagar et al. describes the “dolichomesocolic colon,” one that is vertically longer than it is wide. This anatomic configuration is more commonly found in male subjects and people over the age of 30
The incidence of sigmoid volvulus peaks in the mid-70s for both men and women suggesting that this process is secondary to chronic changes
Sigmoid volvulus is also the most common cause of intestinal obstruction in pregnancy, accounting for 45% of obstructions in this group. Higher rate of misdiagnosis with maternal mortality reported at 5% but up to 50% in the setting of perforation. Fetal demise approaches 30% and is secondary to reduction in placental blood flow due to mass effect from a massively dilated sigmoid.
Plain Radiography: coffee bean”sign or “omega loop with the apex pointing to the right upper abdomen is present in 60% of cases
Barium Enema: bird’s beak” appearance
bird’s beak” sign in combination with characteristic findings on plain radiography carries a diagnostic accuracy of 100%
Computed Tomography:
accuracy approaching 100% (mesenteric twist being highly accurate for the diagnosis of volvulus)
sigmoid transition point is the most sensitive
X-marks-the-spot” sign and the “split wall” sign are also suggestive of sigmoid volvulus and represent a complete twist of adjacent sigmoid loops in a “twist-tie. These findings, though diagnostic, are seen in less than 50% of cases.
- HVPG, False statement?
- 10 mmg is threshold for GE varices development
- 12 mmhg is threshold for variceal bleeding
- 16 mm hg: First clinical decompensation in patients with varices
- In decompensated cirrhosis HVPG 20 mmhg is threshold for increased risk of Spontaneous bacterial peritonitis
Answer: D (Blumgart 6e, page 90)
- Criteria for non resectability of Hilar cholangiocarcinoma all except?
- Bilateral hepatic duct involvement up to secondary biliary radicals
- Positive lymph nodes outside the hepatoduodenal ligament
- Hepatic duct involvement up to secondary biliary radicles with atrophy of contralateral hepatic lobe
- Encasement of portal vein branch with ipsilateral atrophy
Answer: D (Blumgart 6e, page 822)
- Hilar cholangiocarcinoma
- 50% of all cholangiocarcinomas
- majority of these malignancies develop sporadically, but certain risk factors for their development exist.
- primary sclerosing cholangitis (PSC): 6-20% lifetime risk
- autoimmune process
- choledochal cysts:6-30% lifetime risk
- Parasitic infections
- Chronic hepatolithiasis
- thorium dioxide (Thorotrast)
- nitrosamines
- Lynch syndrome and
- biliary papillomatosis
- Greater than 90% of patients with hilar cholangiocarcinoma present with obstructive jaundice, although if only the right or left hepatic ducts are involved, patients may not become jaundiced but may still have vague abdominal pain
- Cholangiocarcinoma characteristically spreads in a longitudinal subepithelial manner along the bile ducts, and perineural and lymphovascular invasion are frequently seen (Patel et al, 2011). This longitudinal spread extends in a plane deep to the intact epithelium and may account for as much as 2 cm of proximal spread and 1 cm of distal spread
- Majority of cholangiocarcinomas are adenocarcinomas and are subdivided into three histologic subtypes: sclerosing, nodular, and papillary.
- Sclerosing tumors are the most common subtype, extend in a subepithelial manner, and cause annular thickening of the bile ducts without a prominent intraductal obstructing mass component
- Nodular tumors also project into the lumen of the duct and cause annular thickening of the ducts as well.
- Papillary tumors make up only 10% of cholangiocarcinoma and are more common in the distal bile duct These fleshy polypoid tumors expand the lumen of the duct and have less risk of an invasive component. Papillary tumors are thus more easily resectable and have a better prognosis compared with the sclerosing and nodular subtypes
- Pancreatoblastoma, true statement?
- Mean age: 60 yrs
- Usually small size ( 1-3 cm )
- Not associated with Beckwith-Wiedemann syndrome
- Some cases are associated with elevated serum AFP level
Answer: D (Blumgart 6e, page 948)
- Pancreatoblastoma is an extremely rare childhood tumor of the pancreas (mean age, 4 years), which can also occasionally arise in adults
- Pancreatoblastomas are usually large (7-18 cm).
- Some cases are associated with elevated serum AFP levels, and occasional cases are seen in association with Beckwith-Wiedemann syndrome or familial adenomatous polyposis(FAP) syndrome.
- Pancreatoblastomas are malignant tumors with a 5-year survival of about 25%, although children diagnosed before the development of metastases have had more favorable outcomes. Pancreatoblastomas are fundamentally acinar cell neoplasms.
- Microscopically, they have sheets of primitive-appearing epithelial cells and acinar formations. A diagnostic requirement is the squamoid nest, which is a small morular arrangement of squamoid cells specific for this tumor type in the pancreas
- Pancreatoblastomas exhibit all three lines of pancreatic differentiation—acinar, neuroendocrine, and ductal—with acinar elements being the most consistently present.
- Molecular genetic findings of pancreatoblastomas differ from the findings of ductal adenocarcinoma; they are associated with APC/β-catenin pathway alterations . The squamoid nests (morules) are interesting in this regard because they are also often encountered in other cancer types in which this pathway is abnormal
Solid Pseudopapillary Neoplasm
- Solid pseudopapillary neoplasm is seen predominantly in young women (mean age, 25 years; >80% female
- AKA: papillary cystic tumor, solid and papillary tumor, solid and cystic tumor, and Frantz tumor
- (<15%), metastases are present
- Often relatively large tumors (mean, 8-10 cm).
- The tumor typically expresses
- Vimentin,
- Progesterone receptors,
- CD10,
- CD117,
- Some of the neuroendocrine markers,
- CD56 and
- Synaptophysin
- Chromogranin, the most specific neuroendocrine marker, is negative, which is important for the differential diagnosis with pannets, and pancreatic enzymes are not expressed. Β-Catenin and cyclin D1 expression have been found in these tumors, suggesting an alteration in the WNT signaling pathway, and the abnormal nuclear labeling pattern for β-catenin can be helpful diagnostically. E-cadherin and n-cadherin expression are also abnormal
- All are indications of liver transplantation except:
- Cholangiocarcinoma
- Cirrhosis
- Biliary atresia
- Fulminant hepatitis
Answer: A (Blumgart 6e, page 1737-1738)
Absolute contraindications to liver transplantation include the following:
- The patient is not fit enough to withstand surgery (e.g., because of advanced cardiac or pulmonary disease).
- The patient is unlikely to survive the procedure (e.g., active sepsis).
- Survival after transplantation may be too short to justify the risks of transplantation (e.g., with metastatic disease).
- Active alcoholism or substance misuse is present (noncompliance with medical advice ascertained on psychiatrist or psychologist evaluation).
- The postoperative quality of life may be unacceptable to the patient (e.g.,, severe intractable depression).
- Surgery is technically not possible (e.g., patients with extensive venous thromboses).
- The patient does not choose to undergo the operation.
Relative contraindications to transplantation include the following:
- Age: age limits vary among transplant centers; biologic rather than chronologic age is more relevant but more difficult to define and quantify.
- Obesity: those with body mass index (BMI) greater than 40 kg/m2 have a poorer outcome.
- Cholangiocarcinoma: considered only in highly select patients in agreed protocols with chemotherapy and irradiation because of early recurrence.
- Chronic or refractory active infections: depending on type of infection and whether amenable to cure with available therapy.
- Poor social support despite full interventions that will adversely impact on graft or patient survival, particularly in biologically older patients with anticipated prolonged recovery after transplant.
- Ongoing tobacco use or illegal drug use.
- All of the following are true in case of chronic liver disease except:
- MELD score is used frequently to see whom needs liver transplant early
- MELD score includes serum albumin, creatinine, and INR
- Child Pugh score includes PT-INR, albumin and bilirubin
- Child Pugh score has class A, B, C
Answer: B (Sabiston 20e, page640
MELD) score, which reflects the likelihood of death within 3 months.
The MELD score assigns points that reflect the severity of liver disease.
The “Share 35” rule dictates that regional rather than local distribution of livers be done for patients with the greatest need (MELD score > . The hope is to further decrease death on the waiting list.
For patients with low MELD scores (<15), the risk of death while waiting for transplantation is less than the risk of death after transplantation. The current allocation system therefore discourages transplantation of patients with MELD scores of less than 15 by the region before allowing local use in patients with scores of less than 15. These last two concepts, that sicker patients have a slightly higher risk of poor outcome after liver transplantation and that relatively healthy patients (MELD score < 15) have worse outcomes with transplantation than if they remained on the waiting list, suggest that both pretransplantation and posttransplantation outcomes should be considered in the allocation of livers.
- In gallstone ileus, obstruction is seen at:
- Jejunum
- Proximal ileum
- Distal ileum
- Colon
Answer: C (Bailey 27e, page 1282)
- Elderly
- 10%: multiple
- Impaction: 60 cm proximal to the ileocaecal valve.
- Recurrent attacks incomplete or relapsing : Ball-valve effect
- Riglers tirad (2 signsare pathognomic, seen in 40-50% )
- Si obstruction
- Pneumobilia
- Atypical mineral shadow on radiographs
- “Apple peel bowel with loss of dorsal mesentery is feature of which type of ileal atresia?
- Type 1
- Type 2
- Type 3
- Type 4
Answer: c(Shakelford 8e, page 975)
- Jejunum(51%) =Ileum
- In utero vascular accident: Necrosis & resorption
- Vasoactive medications in pregnancy
- 1 in 2000-3000
- Prenatal Usg:
- Polyhydromnios (50%)
- Post natal
- Biliary emesis
- Abdomen distension
- 1/3rd: pass meconium in 1st 24 hr
- Concomitant anomaly : 1/3rd
- CF: 10-20%
- Xray
- Dilate loops
- Absent distal air
- Contrast enema
- Microcolon
- Small caliber T ileum
- Abrupt end of passage
- Villous polyp of rectum manifest:
- Bleeding PR
- Mucus diarrhea with hypokalemia
- Prolapse rectum
- Obstruction
Answer: B (Bailey 27e, page 1326)
- Chi Square test
- Crosstabulation
- For categorical variables.
- Evaluate Tests of Independence
- P value not determined by this test
Answer: D (statisticssolutions.com/using-chi-square-statistic-in-research/)
- The Chi Square statistic is commonly used for testing relationships between categorical variables
- The Chi-Square statistic is most commonly used to evaluate Tests of Independence when using a crosstabulation (also known as a bivariate table). Crosstabulation presents the distributions of two
- categorical variables simultaneously, with the intersections of the categories of the variables appearing in the cells of the table
Medical Gastro Test
Que 1. Which of the following statement is true
- Recurrent bacterial cholangitis is a common complication of PBC
- “Overlap syndrome” refers to the combination of NASH and hepatitis C.
- Liver biopsy is usually diagnostic in PBC.
- About 70% of cases of AIH occur in women and 90% of cases of PBC occur in women, but about 60%–70% of cases of PSC occur in men.
Ans is D
About 70% of cases of AIH occur in women and 90% of cases of PBC occur in women, but about
60%–70% of cases of PSC occur in men.
“Overlap syndrome” is a poorly defi ned term but usually refers to a clinical setting where features of more
than one autoimmune and/or cholestatic disease are present. Most commonly, overlap syndrome refers to patientswith histological features of AIH, yet are ANA-negative and antimitochondrial antibody-positive (AIH-PBC overlap). Th is is in distinction to “autoimmune cholangiopathy” in which the histological appearance is that of PBC but with negative antimitochondrial antibody and often positive ANA. Some patients respond biochemicallyto ursodeoxycholic acid monotherapy, others to ursodeoxycholic acid and typical AIH agents, and some arerefractory to therapy. Finally, there is a recognized group of patients with histological (and occasionally serological)features of AIH and cholangiographic evidence of sclerosing cholangitis (AIH-PSC overlap).
Recurrent bacterial cholangitis is a common complication of PSC
The florid bile duct lesion, which is essentially diagnostic for PBC, is only seen in a minority of cases.
Usually, the liver biopsy is “consistent with” a diagnosis of PBC.
Que 2. How many major hepatic veins drain into the inferior vena cava?
A.Two
- Three
- Four
- Five
Ans is B
Que 3. A 45-year-old man reports intermittent dysphagia to solids. An upper endoscopy is performed, and the patient is found to have a Schatzki ring. What is the most likely diameter of the esophageal lumen?
- 12 mm
- 14 mm
- 16 mm
- 18 mm
Ans is A
A Schatzki ring is typically symptomatic at 13 mm or less.The obstructing ring can be visualized radiographically
by entrapment of a 13 mm marshmallow or barium tablet. However, the presence of Schatzki ring of any diameter in
a patient with dysphagia warrants endoscopic dilation to disrupt the ring to try to improve the patient’s symptoms.
Que 4. Which of the following cells in the pancreas is responsible for fibrosis in chronic pancreatitis?
- Acinar cells
- Duct cells
- Pancreatic stellate cells
- Alpha cells
Ans is C
The pancreatic stellate cells secrete collagen and other extracellular proteins to preserve the pancreatic microarchitecture
and facilitate acinar sell secretions. These cells get activated during pancreatic injury and provide the necessary
matrix for healing. However, with persistent injury, they can induce significant fibrosis and result in chronic
pancreatitis. The primary role for acinar cells is secretion of digestive enzymes. Duct cells primarily secrete bicarbonate
and water. Alpha cells secrete glucagon. F cells (also called PP cells) secrete pancreatic polypeptide and
adrenomedullin.
Que 5. A 55-year-old man complains of chronic heartburn and has been on chronic PPI therapy. What is the most likely
finding on upper endoscopy?
- Pancreatic rest
- Fundic gland polyp
- Inflammatory gastric polyp
- Hyperplastic polyp
Ans is B
Fundic gland polyps are benign growths that are associated with chronic PPI therapy. They carry a negligible
malignant potential, and are considered incidental findings on routine upper endoscopy
Que 6. Which of the following is considered a risk factor for
cholelithiasis?
- Rapid weight gain
- Ulcerative colitis
- Octreotide
- Gastrostomy tube feeding
ANs is C
Octreotide increases the prevalence of gallstones when used as treatment for acromegaly, which is likely due to
decreased gallbladder motility. Cholelithiasis is not associated with feeding through a gastrostomy, but rather with
total parenteral nutrition. Rapid weight loss, ulcerative colitis, and statins have not been shown to be associated
with cholelithiasis. Other risk factors include obesity, ileal Crohn’s disease, and treatment with octreotide.
Que 7. All are true about GAVE except
- Dilated submucosal venous plexus
- Bleeding is the most common presentation
- Pain is most common clinical symptom
- Argon laser treatment is established one
ANs is C
Watermelon Stomach (Gastric Antral Vascular Ectasia)
- A rare entity characterized by presence of both inflammatory and vascular components in mucosaQ.
- On endoscopy: Prominent longitudinal folds with parallel striking red stripes atop the mucosal folds of the distal
stomach, much like the rind of a watermelonQ.
Pathology:
- Dilated mucosal blood vessels in the lamina propria, often containing thrombi, with no evidence of vascular malformation on
angiographic and morphologic examinationQ.
- Mucosal fibromuscular hyperplasia and hyalinization are often presentQ.
- Predominantly affects the distal portion (Antrum)Q of the stomach
Clinical Features:
- Patients are generally elderly women with chronic bleedingQ.
- Most have an associated autoimmune connective tissue disorders, and at least 25% have chronic liver diseaseQ.
- Patients typically have iron deficiency anemia and chronic blood loss requiring transfusionsQ
Diagnosis:
- The diagnosis is based on typical endoscopic and biopsy appearance of the mucosaQ.
- Gross endoscopic examination reveals prominent longitudinal folds with parallel striking red stripes atop the
mucosal folds of the distal stomach, much like the rind of a watermelonQ.
Treatment:
- The lesions are treated by endoscopic cauteryQ.
- In patients with portal hypertension, TIPS should be considered firstQ.
Que 8. A 35-year-old male patient was admitted with acute severe ulcerative
colitis. He had been on a reducing dose of steroids while at home,
but failed to respond. After 5 days of treatment with intravenous
hydrocortisone he became tachycardic and complained of abdominal
pain.
Investigations:
haemoglobin 136 g/L (115–165)
serum albumin 31 g/L (37–49)
serum C-reactive protein 68 mg/L (< 10)
What is the next most appropriate investigation?
- CMV serum PCR
- CT abdomen
- Faecal calprotectin
- Stool for Clostridium diffi cile
Ans is B
Acute tachycardia with or without abdominal pain (often masked by steroids) is a worrying
feature in acute severe ulcerative colitis. It warrants urgent assessment and investigation for
complications that may require urgent surgery. CT scanning has become the gold standard for
identifying perforation, and where available is preferable to a plain erect chest X-ray and abdominal
fi lm. Clostridium diffi cile is an important consideration, but should already have been tested for by
day 5 of the presentation.
Indications for surgery in the acute setting include the following:
- Failure of medical management
- Perforation
- Toxic megacolon
- Severe haemorrhage
- Malignancy.
The mortality of acute severe colitis is associated with increasing age (> 50 years), male gender,
and the presence of C. diffi cile diarrhoea. Short-term mortality is higher in emergency colectomies
compared with elective colectomy, but nonetheless surgery can be life-saving.
The timing of surgery is often diffi cult, and should involve a comprehensive MDT approach,
including surgeons, stoma nurses, IBD nurses, radiologists, dietitians, pharmacists, and pathologists
where a diagnostic dilemma occurs. These people should be involved early in the admission. The
latest BSG guidelines suggest that the operation of choice for acute severe colitis would be subtotal
colectomy, end ileostomy, and preservation of a long rectal stump, which can be oversewn and
remain in the peritoneal cavity or brought out as a mucous fi stula. It is not recommended that an
ileoanal pouch is created at the initial surgery. Completion of surgery should occur when the patient
is fully recovered, to reduce complications.
Que 9:- Most common site of post ischemic stricture is:
a.Ascending colon
b.Hepatic flexur
c.Splenic flexur
d.Sigmoid colon
Ans is D
Que 10 :- 91. All are true about fibrolamellar HCC except:
a.AFP is not raised
b.Recurrence is common
c.Raised neurotensin levels
d.Well demarcated and encapsulated
Ans is D
Fibrola mellar HCC
- Occurs in young adults without underlying cirrhosisQ
- Non-encapsulated but well circumscribed, so high resectability rateQ
- Grows slowly and and has better prognosisQ
Pathology
- Well demarcated and non-encapsulated and may have a central fibrotic areaQ.
- The central scar can make distinguishing this tumor from FNH difficult.
- Composed of large polygonal tumor cells embedded in a fibrous stroma forming lamellar structures
- CalcificationQ differentiates FHCC from FNH in 35-55% of FHCC; heterogeneous enhancementQ is also an important
imaging finding.
Clinical Features
- Occurs in younger patientsQ without a history of cirrhosis.
- FHCC doesn’t produce AFPQ
- Associated with elevated neurotensinQ and Vitamin B12 binding globulin levelsQ
Treatment
- Better prognosis than HCC due to high resectability rates, lack of chronic liver disease, and a more indolent courseQ.
- Long-term survival can be expected in about 50-75% of patients after complete resectionQ
- Recurrence is common and occurs in at least 80% of patientsQ.
- The presence of lymph node metastases predicts a worse outcomeQ.
Que 11:- A 35-year-old man with multiple allergies presents for
endoscopy with symptoms of heartburn and dysphagia
to solid and liquids. Endoscopic exam is unremarkable.
What is the next best step in management?
- Empirically dilate the esophagus to 13 mm
- Biopsy the distal esophagus to confirm reflux
esophagitis
- Biopsy the distal, mid, and proximal esophagus
- Schedule the patient for pH/impedance testing
Ans is C
The current indication for esophageal biopsies in the evaluation
of GERD is to exclude eosinophilic esophagitis, especially
in those patients who complain of dysphagia. The most
accepted protocol is to take at least two biopsies from the distal,
mid, and proximal esophagus to optimize the sensitivity
to detect eosinophilic esophagitis. A trial of PPI therapy and
scheduling a patient for ambulatory pH/impendence testing
may be considered if the biopsies for eosinophilic esophagitis
are negative. Empiric dilation is not considered a standard
management strategy in GERD without any mucosal abnormalities,
and may increase the risk of perforation in a patient
with untreated eosinophilic esophagitis.
Que 12 ;- What is the most dominant mechanism for gastroesophageal reflux episodes?
A.Increased transient lower esophageal sphincter relaxations
- Swallow-induced lower esophageal sphincter relaxation
- Absent basal lower esophageal sphincter pressure
- Straining
- Acid hypersecretion
Ans is A
Increased frequency of transient lower esophageal sphincter relaxations are the most dominant mechanism for GERD. These relaxations have four characteristic features: (1)occur independently of swallowing, (2) are not associated
with peristalsis, (3) persist longer (greater than 10 seconds),(4) are accompanied by inhibition of the crural diaphragm.
Que 13. Which of the following ulcers is most likely to have the best outcome and lowest risk of re-bleeding?
- A 2.5-cm clean base ulcer at the antrum
- A 1-cm ulcer in the cardia with a flat pigmented spot
- A 1.5-cm clean base ulcer at the fundus
- A 0.5-cm ulcer with pigmented spot at the posterior
duodenal bulb
Ans is C
Size, location, and bleeding stigmata are most important
prognostic factors of the ulcer. According to Forrest classification,
ulcers with flat pigmentation and clean base do
not require endoscopic interventions given low risk of rebleeding
(see table at end of chapter). Ulcer size (>2 cm) and
location (either high on the lesser curvature of the stomach
or in the superior or posterior duodenal bulb) are important
prognostic factors associated with poorer outcomes.
These are locations where ulcers can erode into major arteries, such as the left gastric artery and the gastroduodenal
artery, respectively. These ulcers occur mostly in older
adult patients, often with significant comorbidities, who
often present with hemodynamic instability. Among all
options, choice C is the only one with no high-risk features
Que 14 :- What is the frequency of delayed gastric emptying in
patients with functional dyspepsia?
- <5%
- 5%-10%
- 20%-50%
- 60%-75%
- >75%
Ans is C
The presence of delayed gastric emptying in functional dyspepsia ranges from 20% to 50%. In a meta-analysis of 17 studies involving 868 dyspeptic patients and 397 controls,
a significant delay in solid gastric emptying was
present in almost 40% of patients with functional dyspepsia.
However, studies failed to find a convincing relationship
between delayed gastric emptying and the pattern of
symptoms.
Que 15 :- In Gallstone ileus, obstruction is seen at:
- Jejunum b. Proximal ileum
- Distal ileum d. Colon
Ans is C
Gallstone Ileus
- Passage of a stone through a spontaneous biliary-enteric fistula leading to a mechanical bowel obstructionQ
- MC site of fistula: Between the gallbladder and duodenumQ
- 2nd MC site: Between gallbladder and transverse colonQ.
Clinical Features
- Rigler’s triadQ: The classic plain abdominal film triad of small bowel obstruction, pneumobilia, and ectopic
gallstone is considered pathognomonicQ.
- Most cholecystoduodenal fistula does not result in Gallstone ileusQ. Rather, they are asymptomaticQ or occur in associated with
usual digestive complaints consistent with gastric or biliary tract disease.
Que 16 - Precancerous lesions of GB are all except:
- Porcelain GB
- Typhoid carrier
- ABPDJ
- Biliary ascariasis.
Ans is D
Que 17 :- All are true about pancreatic serous cystadenoma except:
- Increased CEA
- Honey comb appearance
- Lined by single layer of low cuboidal epithelium
- Malignancy is rare
Ans is A
Que 18 :- A 46-year-old teacher presented with a 5-month history of profuse
diarrhoea despite fasting, and associated abdominal bloating.
She complained of increasing fatigue and appeared dehydrated.
Investigations:
haemoglobin 125 g/L (130–180)
serum sodium 138 mmol/L (137–144)
serum potassium 1.9 mmol/L (3.5–4.9)
serum urea 7.3 mmol/L (2.5–7.0)
serum creatinine 105 μmol/L (60–110)
plasma viscosity 1.76 mPa/s (1.50–1.72)
Which of the following is the most likely diagnosis?
- Bile salt malabsorption
- Coeliac disease
- Irritable bowel syndrome
- VIPoma
Ans is D
This patient exhibits the symptoms of watery profuse diarrhoea, abdominal pain, fatigue,
and bloating that are found in VIPoma, or Verner–Morrison syndrome. Patients complain of
high-volume diarrhoea despite fasting and are found to have achlorhydria, hypokalaemia
(serum potassium < 3 mmol/L), hyperglycaemia, and hypercalcaemia with signs of dehydration.
VIPoma results in high levels of vasoactive intestinal peptide (VIP). VIP usually stimulates fl uid and
electrolyte secretion from intestinal epithelium and bile duct cholangiocytes, and thus overactivity
leads to symptoms of VIPoma.
VIPoma is a rare form of cancer that aff ects an estimated 1 in 10 million people per year, and
consists of typically solitary lesions which are found in the tail of the pancreas (up to 75%). They are
usually diagnosed in adults, most commonly in the age range 30–50 years, and are more common in
women than in men.
Diagnosis is made by measuring fasting plasma VIP levels. Treatment is mostly surgical, but up to
50% of cases can be metastatic at the time of diagnosis. VIPomas are extremely responsive to
octreotide, which results in tumour necrosis and inhibition of growth.
Que 19 :- A 72-year-old man was admitted with a 1-day history of melaena. He
had no relevant past medical history and took no regular medications.
What percentage of patients who are admitted with apparent
acute upper gastrointestinal bleeding do not reveal a cause at
initial gastroscopy?
- 10%
- 20%
- 30%
- 40%
- 50%
Ans is B
Peptic ulcer (44%) is the commonest cause of gastrointestinal (GI)
bleeding. Other causes in order of relative frequency are:
- Oesophagitis: 28%
- Gastritis/erosions: 26%
- Erosive duodenitis: 15%
- Varices: 13%
- Portal hypertensive gastropathy: 7%
- Malignancy: 5%
- Mallory–Weiss tear: 5%
- Vascular malformation: 3%
Que 20 :- All of the following are presinusoidal cause of portal hypertension Except
- Idiopathic portal hypertension
- PBC
- Sarcoidosis
- Sinusoidal obstruction syndrome
Ans is D
Uro Test
- Q-1) A patient comes to your OPD with complaint of retention of urine since 6 hours. Your consultant ask you to asses the status of bladder. What will you will do.
- Inspect the patient’s abdomen from foot end
- Bimanual examination
- Percuss from pubic symphysis towards umbilicus
- Percuss from umbilicus towards pubic symphysis
- A-1) 3 Percussion from pubic symphysis towards umbilicus
- Campbell 12th
- Page number-10
- The bladder is located within the pelvis and can be palpable only as the bladder distends to a level above the pubis, typically greater than 150 mL.
- At a volume of approximately 500 mL, the bladder may be visible as a lower midline abdominal mass in thin patients.
- Percussion starting at the level of the pubic symphysis and ascending toward the umbilicus can help determine the level of distention because the pitch may change from dull to resonant beyond the bladder. Ballottement also can aid in palpation of the bladder.
- A bimanual examination is performed to assess the mobility of the bladder and is the standard of care for examination of patients with large bladder tumors post resection.
- Even in the era of CT and MRI, a bimanual examination can improve upon the performance of these images for the prediction of pT3 disease and was found to be an independent predictor of pT3 disease on multivariate analysis
- Q-2) What is the diagnosis
- Normal Retrograde uretheogram
- Stricture at BM junction
- Stricture at Bulbar urethra
- Urethral distraction injury
- A-2) 1 Normal Retrograde uretheogram
- Campbell 12th
- Page number-38-39.
- A retrograde urethrogram is a study meant to evaluate the anterior and posterior urethra.
- Retrograde urethrography may be particularly beneficial in demonstrating the total length of a urethral stricture, which cannot be negotiated by cystoscopy.
- Retrograde urethrography also demonstrates the anatomy of the urethra distal to a stricture, which may not be assessable by voiding cystourethrography.
- Retrograde urethrography may be performed in the office or in the operating room before performing visual internal urethrotomy or formal urethroplasty.
- Technique - A plain film radiograph is obtained before injection of contrast.
- The patient usually is positioned slightly obliquely to allow evaluation of the full length of urethra.
The penis is placed on slight tension.
- A small catheter may be inserted into the fossa navicularis with the balloon inflated to 2 cc with
sterile water. Contrast then is introduced via a catheter-tipped syringe.
- Alternatively, a penile clamp (e.g., Brodney clamp) may be used to occlude the urethra around the catheter
- Q-3) A patient underwent prostatectomy. Post operatively he developed pelvic fluid collection which required USG guided percutaneous drain under local anaesthesia. As per the Clavein dindo classification, it is grade
- II
- IIIa
- IIIb
- Iva
- A-3) 2 IIIa
- Campbell 12th
- Page number-106
- In 1992, Clavien et al. proposed a new grading system for the severity of complications specifically related to surgical treatments.
- This new framework was centered around the risk and invasiveness of the therapy required to address or treat the complication.
- Q-4) Identify the
incision marked as “4”
- Thoracoabdominal
- Flank subcostal
- Foley muscle splitting
- Chevron
- A-4) 3 Foley muscle splitting
- Campbell 12th
- Page number-139,140
- Q-5) Postoperatively a patient developed weak hand grip and decreased palmar sensation. Intra operatively the arm was dislodged off the armrest in pronation. The most likely injured nerve is
- Radial nerve
- Superficial branch of Ulnar nerve
- Deep branch of Ulnar nerve
- Median nerve
- A-5) 4 Median nerve
- Campbell 12th
- Page number-262
- Radial nerve injury can result in wrist drop.
- It usually occurs when the arm is dislodged off the armrest in
supination and not secured properly.
- When the arm is dislodged in pronation, the median nerve is affected resulting in a weak hand grip and decreased palmar sensation.
- Ulnar nerve injuries occur with excessive hyperextension of the forearm and pronation, or when the elbow is flexed at 90 degrees with arms folded across the chest.
- Postoperatively, a weakened grip ensues or a claw hand consisting of extended metacarpophalangeal and flexed interphalangeal joints usually of the fourth and fifth fingers
- Q-6) You want to asses renal function in a neonate. You plan to send the S. creatinine of neonate. After how many days of life the neonatal S. Creatinine fully reflects the neonatal renal function.
- 1 day
- 2 days
- 5 days
- 7 days
- A-6) 4 7 days
- Campbell 12th
- Page number-341
- Prenatal and neonatal GFR is significantly lower than adult GFR levels.
- Because of placental effects, plasma creatinine concentrations in the first 48 hours after delivery reflects maternal rather than neonatal renal function.
- Under normal circumstances, by 7 days of life, creatinine levels fully reflect neonatal renal function and are normally less than 0.5 mg/dL in term infants.
- Subsequently, GFR doubles in term infants over the first 2 weeks of life and increases steadily until reaching adult levels by 2 years of age.
- Q-7) According to the given sonographic image what is the probable diagnosis
- Posterior urethral valve
- B/L VUR
- B/L PUJO
- Megaurethra
- A-7) 1 Posterior urethral valve
- Campbell 12th
- Page number-409
- Q-8) A patient was taken for microsurgical denervation of spermatic cord. All areas are included in trifecta nerve complex except
- Cremasteric muscle fibre
- Perivasal sheath
- Posterior lipomatous sheath
- Cremasteric artery adventitia
- A-8) 4 Cremasteric artery adventitia
- Campbell 12th edition
- Page-1221
- Microsurgical denervation of the spermatic cord (MDSC) was first described by Devine and Schellhammer.
- This procedure involves transecting the nerves in the spermatic cord, while preserving the blood supply and lymphatics to prevent formation of a hydrocele.
- The internal spermatic veins are ligated, and the cremaster muscle and spermatic cord fascia are preserved.
- Three areas identified by Parekatil et al. that had the highest areas of WD were the cremasteric muscle fibers, the perivasal sheath, and the posterior lipomatous tissue; these areas were called the trifecta nerve complex, which is thought to form the anatomic basis for relief from the denervation procedure.
- Q-9) A 20 year old unmarried patient comes to you OPD with complaint of painless ulcer over penis. There is recent history of unprotected intercourse. On examination there is single ulcer which is indurated has clean base. Also there is nontender bilateral non suppurative lymphadenopathy.What is the drug of choice in this patient.
- Acyclovir
- Benzathine penicillin
- Azithromycin
- Doxycycline
- A-9) 2 Benzathine penicillin
- Campbell 12th edition
- Page-1255,1259
- It is a case of primary syphilis.
- Q-10) Identify the following condition
- Lichen planus
- Lichen nitidus
- Lichen simplex
- Lichen sclerosus et atrophicus
- A-10) 4 Lichen sclerosus et atrophicus
- Campbell 12th edition
- Page-1281,1282
- Lichen sclerosus et atrophicus (LS) is a chronic inflammatory disease of unknown cause with a predilection for the external genitalia.
- LS is 6 to 10 times more prevalent
in women than in men.
- It tends to affect older men (>60 years of age) and can be associated with pain during voiding or erection.
- The glans penis and foreskin are usually affected, and in contrast to women, perianal involvement in men is usually absent.
- Preputial scarring from LS can lead to phimosis, and circumcision is usually
curative, although recurrence in the circumcision scar may occur.
- The late stage of this disease is called balanitis xerotica obliterans, which can involve the penile urethra and result in troublesome urethral strictures
- LS has been associated with SCC of the penis and vulva, particularly those variants not associated with human papillomavirus (HPV), and LS may represent a premalignant condition
- Q-11) A patient comes to you with semen analysis report suggestive of decreased sperm motility. Which of these lubricants is associated with minimum impairment of sperm motility
- Fem glide
- Pre-seed
- Replens
- Astroglide
- A-11) 2 Pre-seed
- Campbell 12th edition
- Page-1434
- Lubricants commonly used during sexual activity such as K-Y Jelly, Keri Lotion, Astroglide, and others are associated with impaired sperm motility.
- Saliva should also be considered toxic to sperm.
- Researchers incubated a variety of lubricants with sperm and observed that the isotonic preparation Pre-Seed did not result in a significant decrease in sperm motility or chromatin integrity.
- In that study, FemGlide, Replens, and Astroglide lubricants resulted in a significant decrease in motility, and FemGlide and K-Y Jelly resulted in a significant decline in sperm chromatin quality.
- Laboratory investigators have also provided evidence that use of Pre-Seed during semen collection for analysis does not affect assessment of bulk seminal parameters, sperm membrane functional integrity, levels of ROSs, total antioxidant capacity (TAC), and DNA integrity
- Q-12) A 45 year old male gives history suggestive of ED.You want to rule out vascular causes of ED. Which is test is most reliable in this patient.
- CIS(Combined intracavernosal injection and stimulation
- Penile plethysmography
- Duplex ultrasonography
- Dynamic infusion Cavernosometry and cavernosography
- A-12) 3 Duplex ultrasonography
- Campbell 12th edition
- Page-1521
- The combined intracavernosal injection and stimulation (CIS) test is a first-line evaluation of penile blood flow because of its very basic manner of administration and assessment.
- The test involves the intracavernosal injection of a vasodilatory drug or drugs as a direct pharmacologic stimulus, combined with genital or audiovisual sexual stimulation, and the erectile response is observed and rated by an independent assessor.
- The test is designed to bypass neurologic and hormonal influences involved in the erectile response and allows the clinician to evaluate the vascular status of the penis directly and objectively.
- Duplex ultrasound of the penis after pharmacostimulation or CIS
represents second-line evaluation of penile blood flow.
- However, it is the most reliable and least-invasive diagnostic modality for assessing ED.
- The test adds an imaging dimension and a quantification component to the evaluation of blood flow in the penis distinct from first-line evaluation, which relies solely on the assessor’s judgment
- Q-13) A patient of Ca Penis without palpable adenopathy is to be taken up for Sentinel lymph node biopsy. Where would these nodes be located
- Superolateral to the junction of the saphenous and femoral veins
- Superomedial to the junction of the saphenous and femoral veins
- Inferomedial to the junction of the saphenous and femoral veins
- Inferolateral to the junction of the saphenous and femoral veins
- A-13) 2 Superomedial to the junction of the saphenous and femoral veins
- Campbell 12th edition
- Page-1760
- The earliest route of dissemination from penile carcinoma is metastasis to the regional femoral and iliac nodes.
- The lymphatics of the prepuce form a connecting network that joins with the lymphatics from the skin of the shaft.
- These tributaries drain into the superficial inguinal nodes (the nodes external to the fascia lata).
- The lymphatics of the glans join the lymphatics draining the corporeal bodies, and they form a collar of connecting channels at the base of the penis that drain by way of the superficial nodes.
- The superficial nodes drain to the deep inguinal nodes (those deep to the fascia lata).
- From there, drainage is to the pelvic nodes (external iliac, internal iliac, and obturator).
- Penile lymphangiographic studies demonstrate a consistent pattern of drainage that proceeds from superficial inguinal to deep inguinal to pelvic node sites without evidence of ipsilateral drainage.
- Multiple cross-connections exist at all levels of drainage so that
penile lymphatic drainage is bilateral to both inguinal areas
- The concept of sentinel lymph node biopsy as described by Cabanas is predicated on detailed penile lymphangiographic studies that have demonstrated consistent drainage of the penile lymphatics into a sentinel node or group of nodes located superomedial to the junction of the saphenous and femoral veins in the area of the superficial epigastric vein.
- Q-14) You are doing percutaneous antegrade endopyelotomy for UPJO. Where will you make the incision
- Anterior
- Posterior
- Medial
- Lateral
- A-14) 4 Lateral
- Campbell 12th edition
- Page-1947,1948
- The indications to intervene for any patient with UPJO include the presence of symptoms, progressive or overall impairment of renal function, development of upper tract stones or infection, or, rarely, causal hypertension
- Even with the acceptance of laparoscopic and robotic pyeloplasty, percutaneous endopyelotomy remains appropriate for patients with UPJO and concomitant pyelocalyceal stones requiring percutaneous access, which can then be managed simultaneously.
- Contraindications to a percutaneous endopyelotomy are similar to the contraindications to any endourologic approach and include a long segment (>2 cm) of obstruction, active infection, and untreated coagulopathy
- Q-15) A patient with newly diagnosed HTN, comes to your clinic. You want to screen that patient for Conn syndrome. Which antihypertensive which needs to be stopped prior to screening.
- Captopril
- Prazosine
- Amlodipine
- Eplerenone
- A-15) 4 Eplerenone
- Campbell 12th edition
- Page-2369,2401
- Before screening is initiated, hypokalemia should be corrected and
all contraindicated medications discontinued.
- Although patients can continue the majority of antihypertensive agents during screening, mineralocorticoid receptor antagonists are contraindicated and should be stopped at least 4 weeks before testing.
- Patients requiring these agents for control of severe hypertension should be transitioned to agents, such as α1-receptor blockers or long- acting calcium channel blockers, with minimal effects on screening test results.
- Other antihypertensive agents can alter screening values, but not to the extent that mandates their discontinuation
- Q-16) You are catheterizing a patient of spinal cord injury. As soon as you insert the foley the patient develops headache,hypertension,flushing and sweating over face.What is the level of spinal cord injury in this patient
- Lesion above brain stem
- Lesion above T6
- Lesion between T6 and S2
- Lesion below S2
- A-16) 2 Lesion above T6
- Campbell 12th edition
- Page-2614
- Autonomic hyperreflexia represents an acute massive disordered autonomic (primarily sympathetic) response to specific stimuli in patients with SCI above the cord level of T6 to T8 (the sympathetic outflow).
- It is more common in cervical (60%) than thoracic (20%) spinal cord injuries. Onset after injury is variable—usually soon after spinal shock, but it may occur up to years after injury, and distal spinal cord viability is a prerequisite.
- Symptomatically, autonomic hyperreflexia is a syndrome of exaggerated sympathetic activity in response to stimuli below the level of the lesion.
- The symptoms include pounding headache, hypertension, and flushing and sweating of the face and body above the level of the lesion.
- Bradycardia is a typical accompaniment, although tachycardia or arrhythmia may be present.
- Hypertension may vary in severity.
- The stimuli for this exaggerated response commonly arise from the bladder or rectum and typically involve distention.
- Precipitation may be the result of simple LUT instrumentation, tube change, catheter obstruction, or clot retention, and in such cases the symptoms resolve quickly if the stimulus is withdrawn.
- Additional causes or exacerbating factors may include other upper urinary tract or LUT pathology (e.g., calculi), gastrointestinal pathology, long bone fracture, sexual activity, electrocoagulation, and decubitus ulcers.
- Q-17) You are doing Uterosacral ligament repair for apical prolapse. Where will you place the suture
- Sacral portion of ligament
- Intermediate portion of ligament
- Cervical portion
- Either 1 or 3
- A-17) 2 Intermediate portion of ligament
- Campbell 12th edition
- Page-2790
- Surgical Anatomy of the Uterosacral Ligaments. The ligament is attached broadly to S1-S3, variably to S4.
- It proceeds in a fanlike manner anterolaterally to the cervical os and also onto the proximal portion of the posterior vagina.
- The ligament can be divided into three portions: the sacral, intermediate, and cervical portions.
- Beneath the sacral portion of the ligament runs the superior gluteal vein.
- The ureter has been reported to be near the anterior margin of the uterosacral ligament near the cervix.
- As the ureter courses distally, the distance between the uterosacral ligament and ureter decreases from 4 cm near the sacrum to 0.9 cm near the cervix.
- Because of the fibrous tissues of the ureteral sheath, the ureter is subject to kinking from traction caused by sutures placed in the adjacent tissue.
- If sutures are placed close to the sacrum, there is a risk of entrapping fibers of the sacral plexus trunk of S1-S4.
- This can result in buttock pain, which radiates to the posterior thigh and popliteal fossa
- Ischial spine is the most useful pelvic landmark and that increased S3 nerve injury may result from suture placement beyond the mean distance of 4.6 cm from the ischial spine in the direction of the path of the uterosacral ligament cephalad toward the sacrum.
- Placing sutures in the intermediate portion of the uterosacral ligament appears to be the optimal site because there are fewer structures to be potentially affected and it provides a stable point of fixation.
- There is decreased chance of traction affecting the ureter at this point,
because it is further away from the ureter and its sheath.
- Placing the sutures from lateral to medial will also minimize accidentally
catching either the ureter or its attachments in the stitch
- Q-18) A females complains of leaking of urine per vagina. On double dye test (with oral phenazopyridine and intravesical blue dye instillation), blue disclouration of tampon is seen in bottom portion. What is the probable diagnosis.
- Ureterovaginal fistula
- Vesicovaginal fistula
- Urethrovaginal fistula
- Vesicouterine fistula
- A-18) 3 Urethrovaginal fistula
- Campbell 12th edition
- Page number-2929
- A double dye or tampon test may confirm the diagnosis of urinary fistula, as well as suggesting the possibility of an associated ureterovaginal or urethrovaginal fistula
- A tampon is placed per vagina. Oral phenazopyridine is administered, and vital blue dye is instilled into the bladder.
- If the tampon is discolored yellow-orange at the top, it is suggestive of a ureterovaginal fistula.
- Blue discoloration in the midportion of the tampon suggests VVF.
- Whereas blue staining at the bottom suggests a urethrovaginal fistula.
- Q-19) You perform transurethral resection of bladder tumour. Pathological reports are suggestive of T1 disease.No muscle tissue was identified in specimen. What will you do next
- Intravesical thepary
- Photodynamic therapy
- Repeat TURBT after 6 weeks
- Cystectomy
- A-19) 3 Repeat TURBT after 6 weeks
- Campbell 12th edition
- Page-3096
- Complete tumor removal is not always possible, whether as a result of excessive tumor volume, anatomic inaccessibility, medical instability requiring premature cessation of initial resection, or risk of perforation.
- It is important to obtain adequate muscle in the biopsy specimen to evaluate muscle invasion, particularly in cases in which T1 disease is identified or invasive tumor is suspected.
- Q-20) A patient of Ca prostate belongs to unfavourable intermediate risk group as per NCCN. What are the recommendations for bone imaging in this patient
- Not needed
- Needed if tumour is T2
- Needed if PSA> 10ng/ml
- Needed when both T2 and PSA>10ng/ml are present
- A-20) 4 Needed when both T2 and PSA>10ng/ml are present
- Campbell 12th edition
- Page number-3526
- A joint guideline from the AUA/ASTRO/SUO for clinically localized prostate cancer risk stratifies men with prostate cancer into very low, low, intermediate, and high risk.
- In the setting of high-grade T1 tumors, repeat TURBT is recommended within 6 weeks of initial TURBT based on the wellestablished risk of identifying worse prognostic findings or upstaging to muscle-invasive disease in up to 25% to 30% of repeat TURBT specimens.
- This is especially important if no muscle is identified on initial pathology, where repeat resection of patients with T1 disease can identify upstaging to muscle- invasive disease in up to 49% of cases